Prove $G/(Mcap N) cong M/(Mcap N) times N/(Mcap N)$ where $G=MN$ and $M,Ntriangleleft G$












2












$begingroup$


If we consider the map $phi: Mtimes N rightarrow M/(Mcap N) times N/(Mcap N)$, I was able to show that this is onto and the kernel of the map is $(Mcap N) times (Mcap N)$ and hence by first isom. theorem,
$$(Mtimes N)/bigl((Mcap N)times (Mcap N)bigr) cong bigl(M/(Mcap N)bigr) times bigl(N/(Mcap N)bigr).$$
But how would I go about showing
$$(Mtimes N)/bigl((Mcap N)times (Mcap N)bigr) cong MN/(Mcap N)$$ to prove the final result? Appreciate your help.










share|cite|improve this question











$endgroup$












  • $begingroup$
    You can do it, but it'll take some work because you need to go from $Mtimes N$ to $MN= G$ first. Better is to consider the maps from $G=MN$ to $G/N$ and to $G/M$, use them to obtain a map to $(G/N)times (G/M)$, and then use the isomorphism theorem that tells you that $HK/K cong H/Hcap K$.
    $endgroup$
    – Arturo Magidin
    Dec 3 '18 at 19:41












  • $begingroup$
    Didn't you just ask this? math.stackexchange.com/questions/3023760
    $endgroup$
    – Arturo Magidin
    Dec 3 '18 at 19:44










  • $begingroup$
    @ArturoMagidin Reading the answers there, it appears that this is a follow up question to attempting to follow one of the answers there. Nonetheless, the previous question should have been linked, and the OP should have made clear that this is a distinct follow up question.
    $endgroup$
    – jgon
    Dec 3 '18 at 21:57
















2












$begingroup$


If we consider the map $phi: Mtimes N rightarrow M/(Mcap N) times N/(Mcap N)$, I was able to show that this is onto and the kernel of the map is $(Mcap N) times (Mcap N)$ and hence by first isom. theorem,
$$(Mtimes N)/bigl((Mcap N)times (Mcap N)bigr) cong bigl(M/(Mcap N)bigr) times bigl(N/(Mcap N)bigr).$$
But how would I go about showing
$$(Mtimes N)/bigl((Mcap N)times (Mcap N)bigr) cong MN/(Mcap N)$$ to prove the final result? Appreciate your help.










share|cite|improve this question











$endgroup$












  • $begingroup$
    You can do it, but it'll take some work because you need to go from $Mtimes N$ to $MN= G$ first. Better is to consider the maps from $G=MN$ to $G/N$ and to $G/M$, use them to obtain a map to $(G/N)times (G/M)$, and then use the isomorphism theorem that tells you that $HK/K cong H/Hcap K$.
    $endgroup$
    – Arturo Magidin
    Dec 3 '18 at 19:41












  • $begingroup$
    Didn't you just ask this? math.stackexchange.com/questions/3023760
    $endgroup$
    – Arturo Magidin
    Dec 3 '18 at 19:44










  • $begingroup$
    @ArturoMagidin Reading the answers there, it appears that this is a follow up question to attempting to follow one of the answers there. Nonetheless, the previous question should have been linked, and the OP should have made clear that this is a distinct follow up question.
    $endgroup$
    – jgon
    Dec 3 '18 at 21:57














2












2








2





$begingroup$


If we consider the map $phi: Mtimes N rightarrow M/(Mcap N) times N/(Mcap N)$, I was able to show that this is onto and the kernel of the map is $(Mcap N) times (Mcap N)$ and hence by first isom. theorem,
$$(Mtimes N)/bigl((Mcap N)times (Mcap N)bigr) cong bigl(M/(Mcap N)bigr) times bigl(N/(Mcap N)bigr).$$
But how would I go about showing
$$(Mtimes N)/bigl((Mcap N)times (Mcap N)bigr) cong MN/(Mcap N)$$ to prove the final result? Appreciate your help.










share|cite|improve this question











$endgroup$




If we consider the map $phi: Mtimes N rightarrow M/(Mcap N) times N/(Mcap N)$, I was able to show that this is onto and the kernel of the map is $(Mcap N) times (Mcap N)$ and hence by first isom. theorem,
$$(Mtimes N)/bigl((Mcap N)times (Mcap N)bigr) cong bigl(M/(Mcap N)bigr) times bigl(N/(Mcap N)bigr).$$
But how would I go about showing
$$(Mtimes N)/bigl((Mcap N)times (Mcap N)bigr) cong MN/(Mcap N)$$ to prove the final result? Appreciate your help.







abstract-algebra group-theory normal-subgroups group-isomorphism






share|cite|improve this question















share|cite|improve this question













share|cite|improve this question




share|cite|improve this question








edited Dec 4 '18 at 0:00









Andrews

3831317




3831317










asked Dec 3 '18 at 19:00









manifoldedmanifolded

615




615












  • $begingroup$
    You can do it, but it'll take some work because you need to go from $Mtimes N$ to $MN= G$ first. Better is to consider the maps from $G=MN$ to $G/N$ and to $G/M$, use them to obtain a map to $(G/N)times (G/M)$, and then use the isomorphism theorem that tells you that $HK/K cong H/Hcap K$.
    $endgroup$
    – Arturo Magidin
    Dec 3 '18 at 19:41












  • $begingroup$
    Didn't you just ask this? math.stackexchange.com/questions/3023760
    $endgroup$
    – Arturo Magidin
    Dec 3 '18 at 19:44










  • $begingroup$
    @ArturoMagidin Reading the answers there, it appears that this is a follow up question to attempting to follow one of the answers there. Nonetheless, the previous question should have been linked, and the OP should have made clear that this is a distinct follow up question.
    $endgroup$
    – jgon
    Dec 3 '18 at 21:57


















  • $begingroup$
    You can do it, but it'll take some work because you need to go from $Mtimes N$ to $MN= G$ first. Better is to consider the maps from $G=MN$ to $G/N$ and to $G/M$, use them to obtain a map to $(G/N)times (G/M)$, and then use the isomorphism theorem that tells you that $HK/K cong H/Hcap K$.
    $endgroup$
    – Arturo Magidin
    Dec 3 '18 at 19:41












  • $begingroup$
    Didn't you just ask this? math.stackexchange.com/questions/3023760
    $endgroup$
    – Arturo Magidin
    Dec 3 '18 at 19:44










  • $begingroup$
    @ArturoMagidin Reading the answers there, it appears that this is a follow up question to attempting to follow one of the answers there. Nonetheless, the previous question should have been linked, and the OP should have made clear that this is a distinct follow up question.
    $endgroup$
    – jgon
    Dec 3 '18 at 21:57
















$begingroup$
You can do it, but it'll take some work because you need to go from $Mtimes N$ to $MN= G$ first. Better is to consider the maps from $G=MN$ to $G/N$ and to $G/M$, use them to obtain a map to $(G/N)times (G/M)$, and then use the isomorphism theorem that tells you that $HK/K cong H/Hcap K$.
$endgroup$
– Arturo Magidin
Dec 3 '18 at 19:41






$begingroup$
You can do it, but it'll take some work because you need to go from $Mtimes N$ to $MN= G$ first. Better is to consider the maps from $G=MN$ to $G/N$ and to $G/M$, use them to obtain a map to $(G/N)times (G/M)$, and then use the isomorphism theorem that tells you that $HK/K cong H/Hcap K$.
$endgroup$
– Arturo Magidin
Dec 3 '18 at 19:41














$begingroup$
Didn't you just ask this? math.stackexchange.com/questions/3023760
$endgroup$
– Arturo Magidin
Dec 3 '18 at 19:44




$begingroup$
Didn't you just ask this? math.stackexchange.com/questions/3023760
$endgroup$
– Arturo Magidin
Dec 3 '18 at 19:44












$begingroup$
@ArturoMagidin Reading the answers there, it appears that this is a follow up question to attempting to follow one of the answers there. Nonetheless, the previous question should have been linked, and the OP should have made clear that this is a distinct follow up question.
$endgroup$
– jgon
Dec 3 '18 at 21:57




$begingroup$
@ArturoMagidin Reading the answers there, it appears that this is a follow up question to attempting to follow one of the answers there. Nonetheless, the previous question should have been linked, and the OP should have made clear that this is a distinct follow up question.
$endgroup$
– jgon
Dec 3 '18 at 21:57










1 Answer
1






active

oldest

votes


















1












$begingroup$

Use first isomorphism theorem again. Can you show that $Mtimes Nto G/(Mcap N)$ given by $(m,n)mapsto mn(Mcap N)$ is a group homomorphism and has kernel $(Mcap N)times (Mcap N)$?



Second edit after reading comments on Servaes' answer on the first question you asked related to this Servaes appears to have suggested this homomorphism in a prior version of his answer according to the comments below it and a skim of revision history. Since you couldn't see why it is a homomorphism, I'll add the following hint: Using that $M$ and $N$ are normal, can you show that the element $mnm^{-1}n^{-1}$ is in $Mcap N$ when $min M$, $nin N$. Can you see that this implies that $M/Mcap N$ and $N/Mcap N$ commute with each other in $G/Mcap N$?



Edit Or alternatively you can follow Arturo's hint.






share|cite|improve this answer











$endgroup$













    Your Answer





    StackExchange.ifUsing("editor", function () {
    return StackExchange.using("mathjaxEditing", function () {
    StackExchange.MarkdownEditor.creationCallbacks.add(function (editor, postfix) {
    StackExchange.mathjaxEditing.prepareWmdForMathJax(editor, postfix, [["$", "$"], ["\\(","\\)"]]);
    });
    });
    }, "mathjax-editing");

    StackExchange.ready(function() {
    var channelOptions = {
    tags: "".split(" "),
    id: "69"
    };
    initTagRenderer("".split(" "), "".split(" "), channelOptions);

    StackExchange.using("externalEditor", function() {
    // Have to fire editor after snippets, if snippets enabled
    if (StackExchange.settings.snippets.snippetsEnabled) {
    StackExchange.using("snippets", function() {
    createEditor();
    });
    }
    else {
    createEditor();
    }
    });

    function createEditor() {
    StackExchange.prepareEditor({
    heartbeatType: 'answer',
    autoActivateHeartbeat: false,
    convertImagesToLinks: true,
    noModals: true,
    showLowRepImageUploadWarning: true,
    reputationToPostImages: 10,
    bindNavPrevention: true,
    postfix: "",
    imageUploader: {
    brandingHtml: "Powered by u003ca class="icon-imgur-white" href="https://imgur.com/"u003eu003c/au003e",
    contentPolicyHtml: "User contributions licensed under u003ca href="https://creativecommons.org/licenses/by-sa/3.0/"u003ecc by-sa 3.0 with attribution requiredu003c/au003e u003ca href="https://stackoverflow.com/legal/content-policy"u003e(content policy)u003c/au003e",
    allowUrls: true
    },
    noCode: true, onDemand: true,
    discardSelector: ".discard-answer"
    ,immediatelyShowMarkdownHelp:true
    });


    }
    });














    draft saved

    draft discarded


















    StackExchange.ready(
    function () {
    StackExchange.openid.initPostLogin('.new-post-login', 'https%3a%2f%2fmath.stackexchange.com%2fquestions%2f3024514%2fprove-g-m-cap-n-cong-m-m-cap-n-times-n-m-cap-n-where-g-mn-and-m-n-t%23new-answer', 'question_page');
    }
    );

    Post as a guest















    Required, but never shown

























    1 Answer
    1






    active

    oldest

    votes








    1 Answer
    1






    active

    oldest

    votes









    active

    oldest

    votes






    active

    oldest

    votes









    1












    $begingroup$

    Use first isomorphism theorem again. Can you show that $Mtimes Nto G/(Mcap N)$ given by $(m,n)mapsto mn(Mcap N)$ is a group homomorphism and has kernel $(Mcap N)times (Mcap N)$?



    Second edit after reading comments on Servaes' answer on the first question you asked related to this Servaes appears to have suggested this homomorphism in a prior version of his answer according to the comments below it and a skim of revision history. Since you couldn't see why it is a homomorphism, I'll add the following hint: Using that $M$ and $N$ are normal, can you show that the element $mnm^{-1}n^{-1}$ is in $Mcap N$ when $min M$, $nin N$. Can you see that this implies that $M/Mcap N$ and $N/Mcap N$ commute with each other in $G/Mcap N$?



    Edit Or alternatively you can follow Arturo's hint.






    share|cite|improve this answer











    $endgroup$


















      1












      $begingroup$

      Use first isomorphism theorem again. Can you show that $Mtimes Nto G/(Mcap N)$ given by $(m,n)mapsto mn(Mcap N)$ is a group homomorphism and has kernel $(Mcap N)times (Mcap N)$?



      Second edit after reading comments on Servaes' answer on the first question you asked related to this Servaes appears to have suggested this homomorphism in a prior version of his answer according to the comments below it and a skim of revision history. Since you couldn't see why it is a homomorphism, I'll add the following hint: Using that $M$ and $N$ are normal, can you show that the element $mnm^{-1}n^{-1}$ is in $Mcap N$ when $min M$, $nin N$. Can you see that this implies that $M/Mcap N$ and $N/Mcap N$ commute with each other in $G/Mcap N$?



      Edit Or alternatively you can follow Arturo's hint.






      share|cite|improve this answer











      $endgroup$
















        1












        1








        1





        $begingroup$

        Use first isomorphism theorem again. Can you show that $Mtimes Nto G/(Mcap N)$ given by $(m,n)mapsto mn(Mcap N)$ is a group homomorphism and has kernel $(Mcap N)times (Mcap N)$?



        Second edit after reading comments on Servaes' answer on the first question you asked related to this Servaes appears to have suggested this homomorphism in a prior version of his answer according to the comments below it and a skim of revision history. Since you couldn't see why it is a homomorphism, I'll add the following hint: Using that $M$ and $N$ are normal, can you show that the element $mnm^{-1}n^{-1}$ is in $Mcap N$ when $min M$, $nin N$. Can you see that this implies that $M/Mcap N$ and $N/Mcap N$ commute with each other in $G/Mcap N$?



        Edit Or alternatively you can follow Arturo's hint.






        share|cite|improve this answer











        $endgroup$



        Use first isomorphism theorem again. Can you show that $Mtimes Nto G/(Mcap N)$ given by $(m,n)mapsto mn(Mcap N)$ is a group homomorphism and has kernel $(Mcap N)times (Mcap N)$?



        Second edit after reading comments on Servaes' answer on the first question you asked related to this Servaes appears to have suggested this homomorphism in a prior version of his answer according to the comments below it and a skim of revision history. Since you couldn't see why it is a homomorphism, I'll add the following hint: Using that $M$ and $N$ are normal, can you show that the element $mnm^{-1}n^{-1}$ is in $Mcap N$ when $min M$, $nin N$. Can you see that this implies that $M/Mcap N$ and $N/Mcap N$ commute with each other in $G/Mcap N$?



        Edit Or alternatively you can follow Arturo's hint.







        share|cite|improve this answer














        share|cite|improve this answer



        share|cite|improve this answer








        edited Dec 3 '18 at 22:07

























        answered Dec 3 '18 at 19:43









        jgonjgon

        13.5k22041




        13.5k22041






























            draft saved

            draft discarded




















































            Thanks for contributing an answer to Mathematics Stack Exchange!


            • Please be sure to answer the question. Provide details and share your research!

            But avoid



            • Asking for help, clarification, or responding to other answers.

            • Making statements based on opinion; back them up with references or personal experience.


            Use MathJax to format equations. MathJax reference.


            To learn more, see our tips on writing great answers.




            draft saved


            draft discarded














            StackExchange.ready(
            function () {
            StackExchange.openid.initPostLogin('.new-post-login', 'https%3a%2f%2fmath.stackexchange.com%2fquestions%2f3024514%2fprove-g-m-cap-n-cong-m-m-cap-n-times-n-m-cap-n-where-g-mn-and-m-n-t%23new-answer', 'question_page');
            }
            );

            Post as a guest















            Required, but never shown





















































            Required, but never shown














            Required, but never shown












            Required, but never shown







            Required, but never shown

































            Required, but never shown














            Required, but never shown












            Required, but never shown







            Required, but never shown







            Popular posts from this blog

            Quarter-circle Tiles

            build a pushdown automaton that recognizes the reverse language of a given pushdown automaton?

            Mont Emei